There are currently 441 dairy cows at Dancing Dairy Farm. Due to some considerations, the number of dairy cows is decreasing at the rate of 13 cows per year. Currently, each cow produces an average of 1157 gallons of milk per year, and production of milk is increasing at the rate of 39 gallons per cow per year. Use the product rule to determine the rate at which milk production at Dancing Dairy Farm is currently changing.

Answers

Answer 1

Answer:

the production of milk at the Dancing Dairy Farm is increasing by 2158 gallons/year

Step-by-step explanation:

Given the data in the question;

Let us represent the number of cows at the farm with x and

each cow produces y gallons of milk

Total mil production will be T.

so

T = x × y

now, differentiating with respect to t

dT/dt = d( xy )/dt

dT/dt = xdy/dt + ydx/dt

given that;

x = 441

y = 1157

dx/dt = -13

dy/dt = 39

so we substitute

dT/dt = ( 441 )( 39 ) + ( 1157 )( -13 )

dT/dt = 17199 - 15041

dt/dT = 2158

Therefore, the production of milk at the Dancing Dairy Farm is increasing by 2158 gallons/year


Related Questions

The length of a rectangle should be 20 meters longer than 8 times the width. If the length must be
between 116 and 180 meters long, what are the restrictions for the width, z?
Write the solution set as an algebraic inequality solved for the variable

Answers

Answer:

The width of the rectangle lies between 12 and 20.

Step-by-step explanation:

Let the width of the rectangle is w.

length of the rectangle,

L = 8 w + 20

116 < L < 180

So,

116 < 8 w + 20 < 180

96 < 8 w < 160

12 < w < 20

So, the width of the rectangle lies between 12 and 20.

the graph below shows how money has been budgeted for a certain company




The company has budgeted approximately 25% of its money on which of the following










Answers

Answer:

Shelter from a tornado should be sought:

in any of the above

in a room in the center of the first floor

in a basement

under a staircase in the center of the house

Shelter from a tornado should be sought:

in any of the above

in a room in the center of the first floor

in a basement

under a staircase in the center of the house

Pls help I’m need to get my grade up

Answers

Answer:

38

Step-by-step explanation:

So, we know the formula is:

D=rt or D=r*t

We only need 2 of the 3 sets of values given in the table, one to find our answer, and the other to double check our answer.

Here are the two sets we can look at:

t=2, d=76

t=3, d=114

Lets plug these in and solve:

76=r*2

Divide both sides by 2 to get r alone:

38=r

Now lets check if this is true by pluggin in 38 for r in the second set, and seeing if it works:

D=r*t

114=38*3

=

114=114

So 38 is our answer.

Hope this helps!

The correct number for your question is 38

A ball is thrown vertically upward from the top of a building. The height (in meters) of the ball after t seconds is given by the function
s(t) = -(t-3)^2+ 14. Find the instantaneous velocity of the ball at t= 4 seconds by considering the average velocities over the intervals [3.5, 4],
[3.7.4]. [3.9,4]. [3.99, 4], [4, 4.01], [4, 4.1], [4, 4.3], and [4,4.5).
ОА.
1.50 m/sec.
OB.
2.00 m/sec.
Ос.
-2.00 m/sec.
OD. -1.50 m/sec.

Answers

9514 1404 393

Answer:

  C.  -2.00 m/sec

Step-by-step explanation:

The average velocity on the interval [a, b] is found by ...

  m = (s(b) -s(a))/(b -a)

One end of the interval remains constant here, so we can define 'd' so that the interval is [4, 4+d]. Then the average velocity is ...

  m = (s(4 +d) -s(4))/((4 +d) -4)

  m = (s(4+d) -s(4))/d

The attached table shows the average velocity values on the intervals required by the problem statement. Respectively, they are ...

  -1.5 m/s, -1.7 m/s, -1.9 m/s, -1.99 m/s, 2.01 m/s, 2.1 m/s, 2.3 m/s, 2.5 m/s

We expect the instantaneous velocity at d=0 to be the average of the values at d=-0.01 and d=+0.01. We estimate the instantaneous velocity at t=4 seconds to be -2.00 m/s.

NO LINKS OR ANSWERING QUESTIONS YOU DON'T KNOW. Use the following information for problems 19-21​

Answers

12 red + 16 blue + 18 white = 46 marbles

9. 3 marbles NO replacement: no white?
28/46 x 27/45 x 26/44
0.61 x 0.6 x 0.59 is about 22%

Each time we pick a red or blue we have one less to choose from next time

10. 4 marbles WITH replacement: all white?
18/46 x 18/46 x 18/46 x 18/46
0.39 x 0.39 x 0.39 x 0.39 is about 2%

11. 2 marbles NO replacement: red then blue

12/46 x 16/45
12 x 45 / 16 x 46
540/736
135/184 :)

en un bolillero hay 10 bolitas iguales numeradas del 0 al 9 ¿cual es el espacio muestral?​

Answers

I don’t know this language

PLS HELP ITS DUE AT 11:10

Answers

Answer:

Scalene.

Step-by-step explanation:

It's scalene because non of the triangle's sides are congruent.

a system of regular payments for when something bad happens
a. Directly b. Reasonable c. Insurance
d. Tuition

Answers

the answer is b. insurance

A final statistics exam had a mean of 70 and a variance of 25. If Bruce earned an 80 on his exam, what is his score?

Answers

Answer:

2

Step-by-step explanation:

Value = 80

Mean = 70

Variance = 25

Standard deviation=root(Variance)=root(25)=5

z score = (value - mean)/standard deviation

            = (80-70)/5

            = 10/5

            = 2

z-score using mean 70 and variance 25 is 2

Given :

A final statistics exam had a mean of 70 and a variance of 25

First we find out standard deviation using variance

standard deviation =[tex]\sqrt{variance} =\sqrt{25} =5[/tex]

Now we find out z-score

[tex]z-score =\frac{value - mean}{standard \; deviation }[/tex]

value =80

mean=70 and standard deviation =5

substitute all the values

[tex]z-score = \frac{80-70}{5} =2[/tex]

So, his z-score using mean and variance is 2

Learn more : brainly.com/question/10649097

solve 4(8-2x)=2(7-x)​

Answers

Answer and Step-by-step explanation:

Solve for x.

First, we divide both sides of the equation by 2.

2(8 - 2x) = 7 - x

Distribute the 2.

16 - 4x = 7 - x

Add 4x to and subtract 7 from both sides of the equation.

9 = 3x

Divide by 3 to both sides of the equation.

x = 3 <-- This is the answer.

#teamtrees #PAW (Plant And Water)

Answer:

x = 3

General Formulas and Concepts:

Pre-Algebra

Order of Operations: BPEMDAS

Brackets Parenthesis Exponents Multiplication Division Addition Subtraction Left to Right

Distributive Property

Equality Properties

Multiplication Property of Equality Division Property of Equality Addition Property of Equality Subtraction Property of Equality

Algebra I

Terms/Coefficients

Step-by-step explanation:

Step 1: Define

Identify

4(8 - 2x) = 2(7 - x)

Step 2: Solve for x

[Division Property of Equality] Divide 2 on both sides:                                  2(8 - 2x) = 7 - x[Distributive Property] Distribute 2:                                                                 16 - 4x = 7 - x[Addition Property of Equality] Add x on both sides:                                     16 - 3x = 7[Subtraction Property of Equality] Subtract 16 on both sides:                       -3x = -9[Division Property of Equality] Divide -3 on both sides:                                x = 3

Find the slope of the line that passes through the following points. Simplify answer (-6,8)and(-1,8)

Answers

Answer:

m = 0

General Formulas and Concepts

Pre-Algebra

Order of Operations: BPEMDAS

Brackets Parenthesis Exponents Multiplication Division Addition Subtraction Left to Right

Algebra I

Coordinates (x, y)Slope Formula: [tex]\displaystyle m = \frac{y_2 - y_1}{x_2 - x_1}[/tex]

Step-by-step explanation:

Step 1: Define

Identify points

Point (-6, 8)

Point (-1, 8)

Step 2: Find slope m

Simply plug in the 2 coordinates into the slope formula to find slope m

Substitute in points [Slope Formula]:                                                              [tex]\displaystyle m = \frac{8 - 8}{-1 - -6}[/tex][Fraction] Subtract:                                                                                            [tex]\displaystyle m = \frac{0}{5}[/tex]Divide:                                                                                                                 [tex]\displaystyle m = 0[/tex]

Susan was posting gifts to her family. She weighed three envelopes before posting them. Geace Envelope A Envelope B Envelope C • Envelope A weighed x grams. Envelope B was 50 grams lighter than Envelope A. • Envelope C was three times as heavy as Envelope A. If the total weight of the three envelopes was 840 grams, write an equation in x and solve it to find the weight of Envelope A. (​

Answers

Answer:

5x+50

Step-by-step explanation:

you have one x. An x plus fifty. And 3 mire x's. So 5 x's and a 50.

pleaseeeee solve thissss pleaseeee

Answers

Answer:

a.) 15 feet

b.) 3.25 seconds

c.) 17.1125 feet

d.) 12.5 seconds

Step-by-step explanation:

a

This is just asking for the y intercept

to get this just do h(0)= 15

b

This is asking for the x value of the vertex

solve that through -b/2a

-1.3/(2*-.2)= 3.25

c

This is asking for the y value of the vertex

to solve this plug in the x value from b

-.2*3.25²+1.3*3.25+15= 17.1125

d

This is asking for an x intercept

using the quadratic formula...

[tex]\frac{-b(+-)\sqrt{b^2-4*a*c}}{2a}=\frac{-1.3(+-)\sqrt{1.3^2-4*-.2*15}}{2*-.2}= -6, 12.5[/tex]

*note the (+-) before the radical is ± *

logic will tell us that a negative x intercept doesn't make any sense so we only take the positive value, 12.5

The sum of and is -15/32 and 7/32 is

Answers

it can be -8/32 or -0.25

Answer:

-8/32 or -1/4(if simplified)

Step-by-step explanation:

-15+7 = -8

-8/32 or -1/4

The height of the saddle off horse above the base ofa carousel can be modeled 4t by the equation f-rr) : 12 sin ^ r 42, where I represents seconds after the ride started. I How much time does to take for the horse to complete one cycle of motion and return to its starting height. What is the maximum height and the minimum height of the horse's saddle above the base ofthe carousel

Answers

Answer:

(a) The time to complete 1 cycle and return is 16/3

(b) The minimum height is 30 inches and the maximum is 54 inches

Step-by-step explanation:

Given

[tex]f(t) = 12\sin(\frac{3\pi}{8}t) + 42[/tex]

Solving (a): Time to complete 1 cycle and return

This implies that we calculate the period.  This is calculated using:

[tex]T = \frac{2\pi}{w}[/tex]

Where:

[tex]w =\frac{3\pi}{8}[/tex]

So, we have:

[tex]T = \frac{2\pi}{\frac{3\pi}{8}}[/tex]

[tex]T = \frac{2}{\frac{3}{8}}[/tex]

[tex]T = \frac{2*8}{3}[/tex]

[tex]T = \frac{16}{3}[/tex]

Solving (b): The maximum and the minimum height

To do this, we have:

[tex]-1 \le \sin(\theta) \le 1[/tex]

Which means:

[tex]-1 \le \sin(\frac{3\pi}{8}) \le 1[/tex]

So, the minimum is:

[tex]\sin(\frac{3\pi}{8}) =- 1[/tex]

And the maximum is:

[tex]\sin(\frac{3\pi}{8}) =1[/tex]

Recall that the height is:

[tex]f(t) = 12\sin(\frac{3\pi}{8}t) + 42[/tex]

So, the maximum and the minimum of  are:

[tex]h_{min} =12 * -1 + 42[/tex]

[tex]h_{min} =30[/tex]

and

[tex]h_{max} =12*1+42[/tex]

[tex]h_{max} =54[/tex]

There are 400 animals that live at a zoo. You find that 22 of 65 randomly chosen animals are
monkeys. About how many animals in the entire zoo are likely to be monkeys?

Answers

Answer:

About 135.

Step-by-step explanation:

As the sample is random the number of monkeys likely to be in the zoo

=  (22/65) * 400

= 135.38

13. Find the length of X (in the picture)​

Answers

Answer:

x=2

Step-by-step explanation:

the sides are proportional due the angles being equal

since the hypotonuse is 5 on the bigger one and 2.5 on the small one we can infer there is a ×2 difference

so 4÷2=x

x=2

The answer is x=2 for sure

Which ordered pair is a solution to the system of inequalities? y< 3x y< 5​

Answers

Answer:

Step-by-step explanation:

Split it into two:

Y<3x

Y<5

X = 1

Y = 3

(1,3)

X = 2

Y = 6

(2, 6)

Plot the line up to y = 4.999999999999999999999 and not higher.

Answer:

Plot the line up to y = 4.999999999999999999999 and not higher

A rectangle is measured 6.5 cm by 7.8 cm .
If its actual area is 52.4 square cm , find the percentage error in the area.

Answers

Answer:

3.24%

Step-by-step explanation:

Percentage error =[ (|calculated value  - actual value) / actual value] x 100

calculated area = length x width

6.5 x 7.8 = 50.7

[(|50.7 - 52.4|] / 52.4] x 100 = 3,24

I’m suppose to find the measure of each angle. Thank you

Answers

Answer:

E) π/9

Step-by-step explanation:

The angles are complementary meaning they add up to 90°.

Convert radians to degrees:

7π/18 · 180/π = 70°

Now we know that we need the radian equivalent of 20°.

Convert degrees to radians:

70 · π/180 = π/9

Therefore, the measure of the missing degree is π/9.

Plz help with this question I don’t get it is an image

Answers

Answer:

D

Step-by-step explanation:

For this question, we know that x represents the thousands of square feet in the stadium. We also know that the stadium is 18 thousand square feet. Therefore, the amount of thousands of square feet, or x, is 18. We can then plug 18 in for x in the equation, getting

[tex]C(x) = \frac{7250000}{18+60} \\= \frac{7250000}{78}\\= 92948[/tex]

as our cost

How many lakhs are there in 4 million?​

Answers

Now,

1 million = 10 lakhs

4 millions = 10 * 4

= 40 lakhs

There are 40 lakhs in 4 millions.

I hope it's help you...

Mark me as brainliest...

Answer: There are 40 lakhs in 4 million.

Step-by-step explanation:

A lakh is 1,00,000. We can divide 4 million by 1,00,000 to get our answer.

[tex]\frac{4,000,000}{1,00,000}[/tex]

The 5 zeros on the 4 million and on 1,00.000 cancel each other out giving us

[tex]\frac{40}{1} \\40[/tex]

Find the lowest common multiple of 10 and 12

Answers

It is 2


2(5) is 10 2(6) is 12. Lower than that is 1 and that is not viable
The lowest common multiple is 60

Mckenna kept track of the number of miles she rode her horse each day for 2 weeks 3.2.8, 8, 2,8, 8, 10, 4, 0, 0, 0, 0, 0, 0, 0 Part A Find the measures of center and variability that best summarize Mckenna's data. Use the dropdown menus to explain your reasoning, Choose The Choose is the best measure hours, is the best measure of center, so the Chooso... of variability LIVE Part 1 Mckenna says that she typically rides her horse for about 6 hours because the mode is 6. Is Mokenna's conclusion appropriate? Use the dropdown menus to explain your reasoning, Mckenna's conclusion Choose... accurate. The mode of her data Choose... 6 hours. However, Choose... than half of the time, she spends Choose.... than 6 hours riding her horse.

Answers

Answer: 110

Step-by-step explanation:

I did it it’s correct :)

What is the probability of flipping a coin 10 times and getting heads 5 times? Round your answer to the nearest tenth percent.

Answers

The Probability is 1/2 or .5, .5 rounded to the nearest tenth would be irregular because .5 is already in the tenth place but according to the law of rounding since ut is five it would be rounded up giving the whole number 1

a number added to 3 is equal to negative 5, find the number​

Answers

Answer:

-8

Step-by-step explanation:

An equation at represents this situation is:

-5 = 3 + x

Find x by;

-5 - 3 = -8

So x is -8:

-5 = 3 + -8 (TRUE)

Hope this helps

[tex]\sf \bf {\boxed {\mathbb {GIVEN:}}}[/tex]

Sum of the two numbers = [tex]-5[/tex]

One of the number = [tex]3[/tex]

[tex]\sf \bf {\boxed {\mathbb {TO\:FIND:}}}[/tex]

The other number.

[tex]\sf \bf {\boxed {\mathbb {SOLUTION:}}}[/tex]

The other number is [tex]\sf\blue{-8}[/tex].

[tex]\sf \bf {\boxed {\mathbb {STEP-BY-STEP\:\:EXPLANATION:}}}[/tex]

Let the other number be [tex]x[/tex].

As per the question, we have

[tex]Sum \: \: of \: \: the \: \: two \: \: numbers = -5 [/tex]

[tex]➺ \: 3 + x = - 5[/tex]

[tex]➺ \: x = - 5 - 3[/tex]

[tex]➺ \: x = - 8[/tex]

[tex]\sf\purple{Therefore,\:the\:other\:number\:is\:-8.}[/tex]

[tex]\sf \bf {\boxed {\mathbb {TO\:VERIFY :}}}[/tex]

[tex] ➪ \: 3 + ( - 8) = - 5[/tex]

[tex] ➪ \: 3 - 8 = - 5[/tex]

[tex] ➪ \: - 5 = - 5[/tex]

➪ L. H. S. = R. H. S.

[tex]\sf\orange{Hence\:verified.}[/tex]

[tex]\red{\large\qquad \qquad \underline{ \pmb{{ \mathbb{ \maltese \: \: Mystique35\:ヅ}}}}}[/tex]

Suppose you have a regular hexagon with all side lengths equal to1. Prove that if you pick seven points from the interior of the hexagon, there is a pair of two points that are distance at most 1 apart.

Answers

Answer:

Proved

Step-by-step explanation:

Given

[tex]n = 6[/tex] --- sides of hexagon

[tex]l = 1[/tex] --- side length

Required

Prove that for 7 points picked from the interior, 2 points are at most 1 unit apart

1. Draw a hexagon  (see attachment)

2. Divide the hexagon into 6 triangles

3. Select 7 points on the hexagon

You will notice that at least 2 points will be in one of the triangle.

The maximum distance between these two points is 1 unit. This is because

1. The triangle is equilateral (all sides equal)

2. The length of each is 1 unit (in other words, the distance between points, cannot exceed the side length)

Jeans are on sale for 20% off the regular price. Which function would give you the total cost for purchasing a pair of jeans that costs x dollars?

Answers

Answer:

0.8X = Final cost

Step-by-step explanation:

Given that jeans are on sale for 20% off the regular price, to determine which function would give you the total cost for purchasing a pair of jeans that costs X dollars, the following calculation must be performed:

1X = Initial cost of the jeans

20% = 0.2

1 - 0.2 = 0.8

0.8X = Final cost

Finding the Area of a Circle Given the Radius Th It The area in terms of pi isi mi? The approximated value for the area is A circle has a radius of 3 miles. Use the work shown below to identify the area in terms of pi and the approximate area of the circle. Use 3.14 for a and round the answer to the nearest tenth. A = 2 A= T(3 mi) A = 3.14(9 mi) ​

Answers

Answer:

I'd use A = πr^2

The area is 28.3 if we're using 3.14 as pi (rounded to the nearest tenth)

Point B has coordinates (4,2). The x-coordinate of point A is - 1. The distance between point A and
point B is 13 units. What are the possible coordinates of point A?

Answers

Answer:

A (-1,-10) ; A (-1,14)

Step-by-step explanation:

[tex]\sqrt{(-1-4)^2 + (y-2)^2} = 13 \\ 25 + y^2 + 4 -4y = 169[/tex]

y^2 -4y - 140 = 0

Δ/4 = 4 + 140 = 144

y1 = 2 + 12 = 14

y2 = 2 -12= -10

Other Questions
In NOP, the measure of P=90, NP = 35, ON = 37, and PO = 12. What ratio represents the sine of N? Plz help with this question I dont get it is an image Cranial Inc.has newly set up a website where customers can obtain information as well as buy its products.It notices that people who visit its website are instantly interested in the products,and 85% of the visitors end up making purchases.However,it also notices that the number of visitors are very low as they don't find it on their searches very often.Which of the following should Crayola include in its IBP campaign to increase its website's visibility and traffic? A) Pop-under ads B) Interstitial ads C) Buzz marketing D) Search engine optimization AI Tool and Dye issued 8% bonds with a face amount of $160 million on January 1, 2018. The bonds sold for $150 million. For bonds of similar risk and maturity the market yield was 9%. Upon issuance, AI elected the option to report these bonds at their fair value. On June 30, 2018, the fair value of the bonds was $145 million as determined by their market value on the NASDAQ. Will AI report a gain or will it report a loss when adjusting the bonds to fair value? If the change in fair value is attributable to a change in the interest rate, did the rate increase or decrease? Will the gain or loss be reported in net income or as OCI? What are the x-intercepts of the graphed function? (3, 0) and (0, 1.5)(3, 0) and (1,0)(1, 2) and (1, 0)(0, 1.5) and (1, 0) Five possible motivations for threat actors (i.e., hackers, attackers) who target companies from an information security perspective are wrath, ideology, opportunity, glory and __________. Group of answer choices national defense boredom ease of execution gain What is shunt resistance? How does it help in measuring current? Suppose that F(x)= x^2 and G(x) 4/5 x^2. Which statements best compares the graph of G(x) with the graph of F(x)?A. The graph of G(x) is the graph of F(x) stretched vertically. b. The graph of G(x) is the graph of F(x) stretched vertically and flipped over the x-axis.C. The graph of G(x) is the graph of F(x) compressed vertically.d. The graph of G(x) is the graph of F(x) compressed vertically and flipped over the x-axis. What is the IUPAC name for this organic molecule? Question 3 of 15Match the term in column 1 to the definition in column 2.Privatized?An attempt to eliminate anethnic groupHomogenousA system of legalizedunequal rightsGenocideThe same throughout agroupApartheidOwned by citizens Hormones are chemical molecules produced by glands. Vasopressin is a hormone secreted by the cells of the hypothalamus, a part of the brain. The hormone regulates homeostasis through its effect on the kidney cells. Which statement is true based on this information? Use the Conductivity interactive to identify each aqueous solution as a strong electrolyte, weak electrolyte, or nonelectrolyte. You are currently in a sorting module. Turn off browse mode or quick nav, Tab to items, Space or Enter to pick up, Tab to move, Space or Enter to drop. Strong electrolyte Weak electrolyte NonelectrolyteNH3 NaCl HCI NaOH C12H22O "He likened Tom's death to the senseless slaughter of songbirds by hunters and children."AAtticus; it shows courageBBoo; it shows symbolismDDill; it shows justiceScout; it shows a loss of innocenceEAunt Alexandra; it shows classism Finding the Area of a Circle Given the Radius Th It The area in terms of pi isi mi? The approximated value for the area is A circle has a radius of 3 miles. Use the work shown below to identify the area in terms of pi and the approximate area of the circle. Use 3.14 for a and round the answer to the nearest tenth. A = 2 A= T(3 mi) A = 3.14(9 mi) The pituitary gland is located in the brain and controls __________.A.skin conditionsB.the rest of the endocrine glandsC.the regeneration of nerve cellsD.involuntary motor skillsPlease select the best answer from the choices provided.ABCD help me..........only 23... Socrates states: consider that each shares in clarity to the degree that the subsection it deals with shares in truth. What does he mean? What was the American Civil War. Describe the effects of this important event in the American History. BD is the angle bisector of ABC. Find m ABC and m DBC. (Picture attached!) Thank you!!! True or false: A reduction of the deferred revenue account can be interpreted as a leading indicator of lower future revenues. Explain False. Revenue is recognized when the deferred revenue liability increases. If the deferred revenue account has decreased, more cash came in from customers and more revenue will be recognized in the future. True. Revenue is recognized when the deferred revenue liability decreases. If the deferred revenue account has decreased, less cash came in from customers and less revenue will be recognized in the future. False. Revenue is recognized when the deferred revenue liability decreases. If the deferred revenue account has decreased, less cash came in from customers and more revenue will be recognized in the future. True. Revenue is rec